LSAT and Law School Admissions Forum

Get expert LSAT preparation and law school admissions advice from PowerScore Test Preparation.

 Kdup
  • Posts: 31
  • Joined: Aug 14, 2017
|
#40950
Hi Powerscore,

This particular question gave me some difficulty. I isolated the conclusion which was" This shows that a decrease in humidity can make people ill."... I easily eliminated answer choice C and E. I was then between A, B, and D. I selected answer choice D because it was the most specific and showing that the effects of the decrease in humidity made people ill. Answer Choice A was my other contender...But, I didn't choose it because it did not say that the system changing caused those specific illnesses. I think what was most difficult with this question was the language.... It was not very direct....
 James Finch
PowerScore Staff
  • PowerScore Staff
  • Posts: 943
  • Joined: Sep 06, 2017
|
#40972
Hi K,

With an Assumption question, the easiest way to choose the correct answer between contenders is to utilize the Assumption Negation technique:

Assumption :arrow: Conclusion

Since you've correctly identified the conclusion, you're already halfway there!

Starting with (A), negating it yields:

None of the visits to the school nurse after the system was modified were due to illness :arrow: A decrease in humidity cannot make people ill

Does this work? It seems to, but let's test your other contenders as well.

(B) negates to:

Most of the students at the high school did not suffer from the decrease in air humidity :arrow: A decrease in humidity cannot make people ill

Two issues immediately stand out: the scope is off ("most" versus a definitive "cannot"), and there is nothing about suffering in either the premises or the conclusion, so it doesn't have any effect on the conclusion at all. We can safely eliminate this answer choice.

Moving on to (D):

A decrease of 18% in air humidity does not cause an increase of 25% in one's probability of becoming ill :arrow: A decrease in humidity cannot make people ill

This one is closer to being correct, than (B) was, but the scope is still off. The numbers are too specific to definitively rule out the conclusion, so this answer is incorrect as well.

Hope this clears things up!

Get the most out of your LSAT Prep Plus subscription.

Analyze and track your performance with our Testing and Analytics Package.